2
$\begingroup$

While doing some calculation I came across the following term [$G=G(x,x'),$ $x$ is independent of $x'$] $$K=\frac{\partial G(x,x')}{\partial (\frac{\partial G}{\partial x'})}$$ I tried to think of it as $G=\int_0^{x'}\frac{\partial G(x,y)}{\partial y}dy$ and then i got something like $$K= \int_0^{x'}\frac{\frac{\partial G}{\partial y}}{\frac{\partial G}{\partial x'}}dy$$ $$K=\int_0^{x'}\delta(y,x')dy$$ Is this correct so far, if yes is there any further simplifcation that I can do? Is there any other way to do it? to give further context I was evaluating Poisson bracket $\dot{x'}=\{x', H' \}$ and $H'(x',p')= H(x,p) + \frac{\partial G(x,x')}{\partial t}$. In addition to that we have the relations $p=\frac{\partial G}{\partial x}$ and $p'=-\frac{\partial G}{\partial x'}$

$\endgroup$
4
  • $\begingroup$ You need to be a lot clearer. For example, is $x^\prime=\dfrac{\mathrm dx}{\mathrm dt}$ or something like that? Also, if you want the integral form, you have to have it be $$G(x,x^\prime)=G(x,0)+\int_0^{x^\prime}\frac{\partial G(x,y)}{\partial y}\mathrm dy$$ $\endgroup$ Commented yesterday
  • 1
    $\begingroup$ I think you need to rewrite $G$ as a function such that one of its argument is $\dfrac{\partial G}{\partial x'}$. There is nothing like $\dfrac{\partial G(x,x')}{\partial(\frac{\partial G}{\partial x'})}$. $\endgroup$ Commented yesterday
  • $\begingroup$ $x'$ is just another variable independent of $x$ $\endgroup$ Commented 23 hours ago
  • $\begingroup$ i have added some more info, i hope it clears things out $\endgroup$ Commented 23 hours ago

1 Answer 1

2
$\begingroup$

Actually, you face an implicit change of variable. Indeed, the definition of the variable $y = \frac{\partial G}{\partial x'} = f(x,x')$ establishes a functional relation between $x$, $x'$ and $y$. You might keep $x$ as an independent variable and invert the said functional relation, hence $x' = \phi(x,y)$ and finally $$ K = \frac{\partial}{\partial y}G(x,x') = \left.\frac{\partial G}{\partial x'}\right|_{x'=\phi(x,y)} \cdot \frac{\partial\phi}{\partial y} $$ by the chain rule.

$\endgroup$

You must log in to answer this question.

Start asking to get answers

Find the answer to your question by asking.

Ask question

Explore related questions

See similar questions with these tags.